Stronger than Nesbitt inequality












32












$begingroup$



For $x,y,z >0$, prove that
$$frac{x}{y+z}+frac{y}{z+x}+frac{z}{x+y} geqslant sqrt{frac94+frac32 cdot frac{(y-z)^2}{xy+yz+zx}}$$




Observation:




  1. This inequality is stronger than the famous Nesbitt's Inequality
    $$frac{x}{y+z}+frac{y}{z+x}+frac{z}{x+y} geqslant frac32 $$ for positive $x,y,z$

  2. We have three variables but the symmetry holds only for two variables $y,z$, resulting in a very difficult inequality. Brute force and Largrange Multiplier are too complicated.

  3. The constant $frac32$ is closed to the best constant. Thus, this inequality is very sharp, simple AM-GM estimation did not work.


Update: As point out by Michael Rozenberg, this inequality is still unsolved










share|cite|improve this question











$endgroup$












  • $begingroup$
    Since both sides are already positive, squaring seems to be a good start
    $endgroup$
    – abiessu
    Sep 21 '15 at 1:06










  • $begingroup$
    That's a shame about the bounty - I was hoping that someone would post an answer for this.
    $endgroup$
    – Irregular User
    May 15 '16 at 15:57
















32












$begingroup$



For $x,y,z >0$, prove that
$$frac{x}{y+z}+frac{y}{z+x}+frac{z}{x+y} geqslant sqrt{frac94+frac32 cdot frac{(y-z)^2}{xy+yz+zx}}$$




Observation:




  1. This inequality is stronger than the famous Nesbitt's Inequality
    $$frac{x}{y+z}+frac{y}{z+x}+frac{z}{x+y} geqslant frac32 $$ for positive $x,y,z$

  2. We have three variables but the symmetry holds only for two variables $y,z$, resulting in a very difficult inequality. Brute force and Largrange Multiplier are too complicated.

  3. The constant $frac32$ is closed to the best constant. Thus, this inequality is very sharp, simple AM-GM estimation did not work.


Update: As point out by Michael Rozenberg, this inequality is still unsolved










share|cite|improve this question











$endgroup$












  • $begingroup$
    Since both sides are already positive, squaring seems to be a good start
    $endgroup$
    – abiessu
    Sep 21 '15 at 1:06










  • $begingroup$
    That's a shame about the bounty - I was hoping that someone would post an answer for this.
    $endgroup$
    – Irregular User
    May 15 '16 at 15:57














32












32








32


15



$begingroup$



For $x,y,z >0$, prove that
$$frac{x}{y+z}+frac{y}{z+x}+frac{z}{x+y} geqslant sqrt{frac94+frac32 cdot frac{(y-z)^2}{xy+yz+zx}}$$




Observation:




  1. This inequality is stronger than the famous Nesbitt's Inequality
    $$frac{x}{y+z}+frac{y}{z+x}+frac{z}{x+y} geqslant frac32 $$ for positive $x,y,z$

  2. We have three variables but the symmetry holds only for two variables $y,z$, resulting in a very difficult inequality. Brute force and Largrange Multiplier are too complicated.

  3. The constant $frac32$ is closed to the best constant. Thus, this inequality is very sharp, simple AM-GM estimation did not work.


Update: As point out by Michael Rozenberg, this inequality is still unsolved










share|cite|improve this question











$endgroup$





For $x,y,z >0$, prove that
$$frac{x}{y+z}+frac{y}{z+x}+frac{z}{x+y} geqslant sqrt{frac94+frac32 cdot frac{(y-z)^2}{xy+yz+zx}}$$




Observation:




  1. This inequality is stronger than the famous Nesbitt's Inequality
    $$frac{x}{y+z}+frac{y}{z+x}+frac{z}{x+y} geqslant frac32 $$ for positive $x,y,z$

  2. We have three variables but the symmetry holds only for two variables $y,z$, resulting in a very difficult inequality. Brute force and Largrange Multiplier are too complicated.

  3. The constant $frac32$ is closed to the best constant. Thus, this inequality is very sharp, simple AM-GM estimation did not work.


Update: As point out by Michael Rozenberg, this inequality is still unsolved







inequality






share|cite|improve this question















share|cite|improve this question













share|cite|improve this question




share|cite|improve this question








edited Jan 21 '18 at 3:48







HN_NH

















asked Sep 21 '15 at 0:51









HN_NHHN_NH

83811038




83811038












  • $begingroup$
    Since both sides are already positive, squaring seems to be a good start
    $endgroup$
    – abiessu
    Sep 21 '15 at 1:06










  • $begingroup$
    That's a shame about the bounty - I was hoping that someone would post an answer for this.
    $endgroup$
    – Irregular User
    May 15 '16 at 15:57


















  • $begingroup$
    Since both sides are already positive, squaring seems to be a good start
    $endgroup$
    – abiessu
    Sep 21 '15 at 1:06










  • $begingroup$
    That's a shame about the bounty - I was hoping that someone would post an answer for this.
    $endgroup$
    – Irregular User
    May 15 '16 at 15:57
















$begingroup$
Since both sides are already positive, squaring seems to be a good start
$endgroup$
– abiessu
Sep 21 '15 at 1:06




$begingroup$
Since both sides are already positive, squaring seems to be a good start
$endgroup$
– abiessu
Sep 21 '15 at 1:06












$begingroup$
That's a shame about the bounty - I was hoping that someone would post an answer for this.
$endgroup$
– Irregular User
May 15 '16 at 15:57




$begingroup$
That's a shame about the bounty - I was hoping that someone would post an answer for this.
$endgroup$
– Irregular User
May 15 '16 at 15:57










2 Answers
2






active

oldest

votes


















6












$begingroup$

Here's a proof.



Using Lagrange's identity leads to
$$sum_{cyc}frac{x}{y+z}=frac32+frac12sum_{cyc}frac{(x-y)^2}{(x+z)(y+z)} qquad (*)$$



We get this by letting $a = sqrt{x+y}$, $b = sqrt{y+z}$, $c = sqrt{z+x}$,
$d = frac{1}{sqrt{x+y}}$, $e = frac{1}{sqrt{y+z}}$, $f = frac{1}{sqrt{z+x}}$, and by Lagrange's identity,
$$(a^2+b^2+c^2)(d^2+e^2+f^2) = (ad+be+cf)^2 + (ae-bd)^2+(af-cd)^2+(bf-ce)^2$$
which gives
$$2 (x+y+z)(sum_{cyc}frac{1}{y+z}) = 9 + sum_{cyc} left(frac{sqrt{x+y}}{sqrt{y+z}} - frac{sqrt{y+z}}{sqrt{x+y}}right)^2$$
or
$$3 +sum_{cyc}frac{x}{y+z} = frac92 + frac12sum_{cyc} frac{left({x+y} - (y+z)right)^2}{({y+z})(x+y)} $$
which is the desired equation $(*)$.



Squaring both sides of the inequality then gives



$$
tag{1} frac16 left[ sum_{cyc}frac{(x-y)^2}{(x+z)(y+z)} right ]^2 + sum_{cyc}frac{(x-y)^2}{(x+z)(y+z)} geq frac{(y-z)^2}{xy + yz + xz}
$$



We will follow two paths for separate cases.



Path 1:
Omitting the square term it suffices to prove
$$sum_{cyc}frac{(x-y)^2}{(x+z)(y+z)} geqslant frac{(y-z)^2}{xy + yz + xz}$$



Clearing denominators, we obtain
$$
(x-y)^2 (x+y) + (y-z)^2 (y+z) + (z-x)^2 (z+x) geq frac{(y-z)^2}{xy + yz + xz} (x+y) (y+z) (z+x)
$$



Using $(x+y)(y+z)(x+z) = (x+y+z)(xy + yz+ xz) - x y z$
it suffices to show



$$
(x-y)^2 (x+y) + (y-z)^2 (y+z) + (z-x)^2 (z+x) geq {(y-z)^2} (x+y+z)
$$

or
$$
(x-y)^2 (x+y) + (z-x)^2 (z+x) geq (y-z)^2 x
$$



Since
$$
(y-z)^2 = (y-x + x- z)^2 = (y-x)^2 + (x- z)^2 + 2 (y-x)(x-z)
$$

this translates into



$$
(y-x)^2 y + (x-z)^2 z geq 2 x(y-x)(x-z)
$$

For the two cases $ygeq x ; zgeq x $ and $yleq x ; zleq x $ the RHS $leq 0$ so we are done. For the other two cases, by symmetry, it remains to show the case $y> x ; z < x $.



Rearranging terms, we can also write
$$
(y-x)^3 - (x-z)^3 +x ((y - x) + (z-x))^2 geq 0
$$



This holds true at least for $(y-x)^3 geq (x-z)^3$ or $y+zgeq 2 x$.



So the proof is complete other than for the case $y+z < 2x$ and [ $y> x ; z < x $ or $z> x ; y < x $ ].



Path 2.



For the remaining case $y+z < 2 x $ and [$y> x ; z < x$ or $z> x ; y < x $] we will follow a different path. Again, by symmetry, we must inspect only $y+z < 2x$ and $y> x > z$.



A remark up front: In the following, some high order polynomials of one variable have to be inspected. MATLAB is used, also for plotting behaviours of these polynomials. There is no case in spending effort for further analytical work on polynomials where their behaviour is obvious. Still, what follows contains some "ugly" parts.



In the squared version (1) of the inequality, we can use a further inequality which has been proved here:
$$sum_{cyc}frac{(x-y)^2}{(x+z)(y+z)} geqslant frac{27}{8} frac{(y-z)^2}{(x+y+z)^2}$$



So it suffices to prove



$$
frac16 left[ frac{27}{8} frac{(y-z)^2}{(x+y+z)^2} right ]^2 + sum_{cyc}frac{(x-y)^2}{(x+z)(y+z)} geq frac{(y-z)^2}{xy + yz + xz}
$$



Some numerical inspection shows immediately that the first term cannot be ommitted.



Clearing some denominators gives



$$
tag{2} frac16 left[ frac{27}{8} right ]^2 frac{(y-z)^4 (xy + yz + xz) (x+y)(y+z) (z+x)}{(x+y+z)^4} + \
(xy + yz + xz) sum_{cyc} {(x-y)^2}{(x+y)} - (y-z)^2 (x+y)(y+z) (z+x) geqslant 0 quad
$$



By homogeneity, we set $y=1+z$.



The condition $y+z < 2x$ then translates into $1+2z < 2x$, hence we further set $x = z + (1 +q)/2$ where $0leq q leq 1$ since also $x = z + (1 +q)/2 < y = 1 +z$.



Inserting $y=1+z$ and $x = z + (1 +q)/2$ into (2) is straightforward, the result is lengthy (not displayed here).



Let us start with focussing on the first term in (2), calling that fraction $F$:
$$
F= frac{(y-z)^4 (xy + yz + xz) (x+y)(y+z) (z+x)}{(x+y+z)^4}
$$

With the setting $y=1+z$ this can be simplified to
$$
F = frac{(xy + yz + xz) (x+y)(y+z) (z+x)}{(x+y+z)^4}
$$



Since by the settings $y=1+z$ and $x = z + (1 +q)/2$ both x and y are linear in z, the numerator of $F$ is of fifth order in $z$, whereas the denominator is of fourth order in $z$. In leading order, the whole term is therefore of first order in $z$ and will therefore rise with $z$ for large enough $z$. This motivates to show that indeed
$$F(q,z) geq F(q,z=0) = 2 frac{(q+1)^2}{(q+3)^2}$$
for all $z$ and $q$.
Showing that directly requires a condition



$$
G = (xy + yz + xz) (x+y)(y+z) (z+x) (q+3)^2 - 2 (q+1)^2 (x+y+z)^4 geq 0
$$



Inserting $y=1+z$ and $x = z + (1 +q)/2$ into $G$, and expanding the brackets, gives a very lengthy expression which however contains only positive terms, so the condition is proved immediately:



$$
G = (z(2q^6z + 2q^6 + 22q^5z^2 + 51q^5z + 22q^5 + 80q^4z^3 + 358q^4z^2 + 357q^4z + 100q^4 + 96q^3z^4 + 960q^3z^3 + 1828q^3z^2 + 1206q^3z + 260q^3 + 864q^2z^4 + 3672q^2z^3 + 4788q^2z^2 + 2484q^2z + 450q^2 + 2592qz^4 + 7344qz^3 + 7398qz^2 + 3159qz + 486q + 2592z^4 + 5832z^3 + 4806z^2 + 1701z + 216))/4 geq 0
$$



Hence it suffices, instead of (2), to prove the following:



$$
tag{3} frac16 left[ frac{27}{8} right ]^2 2 frac{(q+1)^2}{(q+3)^2} +
(xy + yz + xz) sum_{cyc} {(x-y)^2}{(x+y)} \ - (x+y)(y+z) (z+x) geqslant 0 quad
$$



After insertion of $y=1+z$ and $x = z + (1 +q)/2$, the factors $(x-y)^2$ in the cyclic sum will not be functions of $z$. Hence,
the LHS is a third order expression in $z$ with leading (in $z^3$ ) term $( 1 + 3 q^2) z^3$, so for large enough $z$ it is rising with $z$. A remarkable feature of this expression is that for the considered range $0leq q leq 1$ it is actually monotonously rising for all $z$. To see this, consider whether there are points with zero slope. The first derivative of the expression with respect to $z$ is



$$
q^4/4 + (7q^3z)/2 + (7q^3)/4 + 9q^2z^2 + 9q^2z + (5q^2)/4 - (7qz)/2 - (7q)/4 + 3z^2 + 3z + 1/2
$$

Equating this to zero gives
$$
z_{1} =
-(2((13q^6)/4 + (17q^4)/2 + (133q^2)/4 + 3)^{(1/2)} - 7q + 18q^2 + 7q^3 + 6)/(36q^2 + 12)\
z_{2} =
-(-2((13q^6)/4 + (17q^4)/2 + (133q^2)/4 + 3)^{(1/2)} - 7q + 18q^2 + 7q^3 + 6)/(36q^2 + 12)
$$

One now shows that in the range $0leq q leq 1$, there are only negative solutions $z_{1,2}$. So we will have no zero slopes for $zgeq 0$. Since the polynomials are (roots of) sixth order in q, we investigate the following figures:



enter image description here



enter image description here



enter image description here



So monotonicity (rising with $z$) is established for all $q$.



Hence, to show the inequality it suffices to inspect (3) at the smallest $z=0$. This gives



$$
((q + 1)(8q^6 + 88q^5 + 336q^4 + 432q^3 - 216q^2 - 405q + 243))/(64(q + 3)^3) geqslant 0 quad
$$

or



$$
tag{4} 8q^6 + 88q^5 + 336q^4 + 432q^3 - 216q^2 - 405q + 243 geqslant 0 quad
$$



An even stronger requirement is
$$
h(q) =
432q^3 - 216q^2 - 405q + 243 geq 0
$$

In the considered range $0leq q leq 1$, $h(q)$ has a minimum which is obtained by taking the first derivative,
$$
1296 q^2 - 432 q - 405
$$

and equating to zero, which gives $q = 3/4$, and the above $h(q)$ then gives
$$
h(q = 3/4) = 0
$$



This establishes the inequality. $ qquad Box$






share|cite|improve this answer











$endgroup$





















    4












    $begingroup$

    Since our inequality is symmetric on $y$ and $z$ , we need to check to cases:




    1. $x=min{x,y,z}$.


    Let $y=x+u$ and $z=x+v$. Hence,
    $$4prod_{cyc}(x+y)^2(xy+xz+yz)left(left(sumlimits_{cyc}frac{x}{y+z}right)^2-frac{9}{4}-frac{3(y-z)^2}{2(xy+xz+yz)}right)=$$
    $$=192(u^2+uv+v^2)x^6+48(10u^3+13u^2v+13uv^2+10v^3)x^5+$$
    $$+48(10u^4+17u^3v+18u^2v^2+17uv^3+10v^4)x^4+$$
    $$+4(62u^5+133u^4v+158u^3v^2+158u^2v^3+133uv^4+62v^5)x^3+$$
    $$+(68u^6+192u^5v+225u^2v^4+298u^3v^3+225u^2v^4+192uv^5+68v^6)x^2+$$
    $$+(8u^7+40u^6v+34u^5v^2+66u^4v^3+66u^3v^4+34u^2v^5+40uv^6+8u^7)x+$$
    $$+uv(u+v)^2(4u^4-6u^3v+11u^2v^2-6uv^3+4v^4)geq0;$$
    2. $xneqmin{x,y,z}$.



    Let $x=y+u$ and $z=y+v$. Hence,
    $$4prod_{cyc}(x+y)^2(xy+xz+yz)left(left(sumlimits_{cyc}frac{x}{y+z}right)^2-frac{9}{4}-frac{3(y-z)^2}{2(xy+xz+yz)}right)=$$
    $$=192(3u^2-3uv+v^2)y^6+48(26u^3-3u^2v-19uv^2+10v^3)y^5+$$
    $$+48(22u^4+21u^3v-26u^2v^2-7uv^3+10v^4)y^4+$$
    $$+4(110u^5+253u^4v-58u^3v^2-226u^2v^3+37uv^4+62v^5)y^3+$$
    $$+(92u^6+384u^5v+201u^2v^4-374u^3v^3-231u^2v^4+144uv^5+68v^6)y^2+$$
    $$+2(4u^7+32u^6v+41u^5v^2-15u^4v^3-51u^3v^4-7u^2v^5+20uv^6+4u^7)y+$$
    $$+uv(u+v)^2(4u^4-u^2v^2-6uv^3+4v^4)geq0.$$
    Done!






    share|cite|improve this answer











    $endgroup$













    • $begingroup$
      great proof! by the way are you some olympiad coach?
      $endgroup$
      – vidyarthi
      Jan 31 '17 at 13:06










    • $begingroup$
      @vidyarthi Yes. I one of Israeli team coaches.
      $endgroup$
      – Michael Rozenberg
      Jan 31 '17 at 13:16










    • $begingroup$
      @MichaelRozenberg Typo in the last line of case 1 where $−6uv^3$ appears twice.
      $endgroup$
      – Andreas
      Feb 8 '17 at 10:15










    • $begingroup$
      @MichaelRozenberg Leaves open a number of smaller proofs to establish that all 14 coefficients of the powers of x and y are nonnegative. This is obvious for 5 coefficients which have only positive summands but not obvious for the other 9.
      $endgroup$
      – Andreas
      Feb 8 '17 at 10:15










    • $begingroup$
      @MichaelRozenberg Dear Michael, my best congratulations to you on having reached 10k reputation today! You have contributed tremendously and even accelerated over the last months! Good to have you around! Very best regards, Andreas
      $endgroup$
      – Andreas
      Feb 8 '17 at 18:33













    Your Answer





    StackExchange.ifUsing("editor", function () {
    return StackExchange.using("mathjaxEditing", function () {
    StackExchange.MarkdownEditor.creationCallbacks.add(function (editor, postfix) {
    StackExchange.mathjaxEditing.prepareWmdForMathJax(editor, postfix, [["$", "$"], ["\\(","\\)"]]);
    });
    });
    }, "mathjax-editing");

    StackExchange.ready(function() {
    var channelOptions = {
    tags: "".split(" "),
    id: "69"
    };
    initTagRenderer("".split(" "), "".split(" "), channelOptions);

    StackExchange.using("externalEditor", function() {
    // Have to fire editor after snippets, if snippets enabled
    if (StackExchange.settings.snippets.snippetsEnabled) {
    StackExchange.using("snippets", function() {
    createEditor();
    });
    }
    else {
    createEditor();
    }
    });

    function createEditor() {
    StackExchange.prepareEditor({
    heartbeatType: 'answer',
    autoActivateHeartbeat: false,
    convertImagesToLinks: true,
    noModals: true,
    showLowRepImageUploadWarning: true,
    reputationToPostImages: 10,
    bindNavPrevention: true,
    postfix: "",
    imageUploader: {
    brandingHtml: "Powered by u003ca class="icon-imgur-white" href="https://imgur.com/"u003eu003c/au003e",
    contentPolicyHtml: "User contributions licensed under u003ca href="https://creativecommons.org/licenses/by-sa/3.0/"u003ecc by-sa 3.0 with attribution requiredu003c/au003e u003ca href="https://stackoverflow.com/legal/content-policy"u003e(content policy)u003c/au003e",
    allowUrls: true
    },
    noCode: true, onDemand: true,
    discardSelector: ".discard-answer"
    ,immediatelyShowMarkdownHelp:true
    });


    }
    });














    draft saved

    draft discarded


















    StackExchange.ready(
    function () {
    StackExchange.openid.initPostLogin('.new-post-login', 'https%3a%2f%2fmath.stackexchange.com%2fquestions%2f1444352%2fstronger-than-nesbitt-inequality%23new-answer', 'question_page');
    }
    );

    Post as a guest















    Required, but never shown

























    2 Answers
    2






    active

    oldest

    votes








    2 Answers
    2






    active

    oldest

    votes









    active

    oldest

    votes






    active

    oldest

    votes









    6












    $begingroup$

    Here's a proof.



    Using Lagrange's identity leads to
    $$sum_{cyc}frac{x}{y+z}=frac32+frac12sum_{cyc}frac{(x-y)^2}{(x+z)(y+z)} qquad (*)$$



    We get this by letting $a = sqrt{x+y}$, $b = sqrt{y+z}$, $c = sqrt{z+x}$,
    $d = frac{1}{sqrt{x+y}}$, $e = frac{1}{sqrt{y+z}}$, $f = frac{1}{sqrt{z+x}}$, and by Lagrange's identity,
    $$(a^2+b^2+c^2)(d^2+e^2+f^2) = (ad+be+cf)^2 + (ae-bd)^2+(af-cd)^2+(bf-ce)^2$$
    which gives
    $$2 (x+y+z)(sum_{cyc}frac{1}{y+z}) = 9 + sum_{cyc} left(frac{sqrt{x+y}}{sqrt{y+z}} - frac{sqrt{y+z}}{sqrt{x+y}}right)^2$$
    or
    $$3 +sum_{cyc}frac{x}{y+z} = frac92 + frac12sum_{cyc} frac{left({x+y} - (y+z)right)^2}{({y+z})(x+y)} $$
    which is the desired equation $(*)$.



    Squaring both sides of the inequality then gives



    $$
    tag{1} frac16 left[ sum_{cyc}frac{(x-y)^2}{(x+z)(y+z)} right ]^2 + sum_{cyc}frac{(x-y)^2}{(x+z)(y+z)} geq frac{(y-z)^2}{xy + yz + xz}
    $$



    We will follow two paths for separate cases.



    Path 1:
    Omitting the square term it suffices to prove
    $$sum_{cyc}frac{(x-y)^2}{(x+z)(y+z)} geqslant frac{(y-z)^2}{xy + yz + xz}$$



    Clearing denominators, we obtain
    $$
    (x-y)^2 (x+y) + (y-z)^2 (y+z) + (z-x)^2 (z+x) geq frac{(y-z)^2}{xy + yz + xz} (x+y) (y+z) (z+x)
    $$



    Using $(x+y)(y+z)(x+z) = (x+y+z)(xy + yz+ xz) - x y z$
    it suffices to show



    $$
    (x-y)^2 (x+y) + (y-z)^2 (y+z) + (z-x)^2 (z+x) geq {(y-z)^2} (x+y+z)
    $$

    or
    $$
    (x-y)^2 (x+y) + (z-x)^2 (z+x) geq (y-z)^2 x
    $$



    Since
    $$
    (y-z)^2 = (y-x + x- z)^2 = (y-x)^2 + (x- z)^2 + 2 (y-x)(x-z)
    $$

    this translates into



    $$
    (y-x)^2 y + (x-z)^2 z geq 2 x(y-x)(x-z)
    $$

    For the two cases $ygeq x ; zgeq x $ and $yleq x ; zleq x $ the RHS $leq 0$ so we are done. For the other two cases, by symmetry, it remains to show the case $y> x ; z < x $.



    Rearranging terms, we can also write
    $$
    (y-x)^3 - (x-z)^3 +x ((y - x) + (z-x))^2 geq 0
    $$



    This holds true at least for $(y-x)^3 geq (x-z)^3$ or $y+zgeq 2 x$.



    So the proof is complete other than for the case $y+z < 2x$ and [ $y> x ; z < x $ or $z> x ; y < x $ ].



    Path 2.



    For the remaining case $y+z < 2 x $ and [$y> x ; z < x$ or $z> x ; y < x $] we will follow a different path. Again, by symmetry, we must inspect only $y+z < 2x$ and $y> x > z$.



    A remark up front: In the following, some high order polynomials of one variable have to be inspected. MATLAB is used, also for plotting behaviours of these polynomials. There is no case in spending effort for further analytical work on polynomials where their behaviour is obvious. Still, what follows contains some "ugly" parts.



    In the squared version (1) of the inequality, we can use a further inequality which has been proved here:
    $$sum_{cyc}frac{(x-y)^2}{(x+z)(y+z)} geqslant frac{27}{8} frac{(y-z)^2}{(x+y+z)^2}$$



    So it suffices to prove



    $$
    frac16 left[ frac{27}{8} frac{(y-z)^2}{(x+y+z)^2} right ]^2 + sum_{cyc}frac{(x-y)^2}{(x+z)(y+z)} geq frac{(y-z)^2}{xy + yz + xz}
    $$



    Some numerical inspection shows immediately that the first term cannot be ommitted.



    Clearing some denominators gives



    $$
    tag{2} frac16 left[ frac{27}{8} right ]^2 frac{(y-z)^4 (xy + yz + xz) (x+y)(y+z) (z+x)}{(x+y+z)^4} + \
    (xy + yz + xz) sum_{cyc} {(x-y)^2}{(x+y)} - (y-z)^2 (x+y)(y+z) (z+x) geqslant 0 quad
    $$



    By homogeneity, we set $y=1+z$.



    The condition $y+z < 2x$ then translates into $1+2z < 2x$, hence we further set $x = z + (1 +q)/2$ where $0leq q leq 1$ since also $x = z + (1 +q)/2 < y = 1 +z$.



    Inserting $y=1+z$ and $x = z + (1 +q)/2$ into (2) is straightforward, the result is lengthy (not displayed here).



    Let us start with focussing on the first term in (2), calling that fraction $F$:
    $$
    F= frac{(y-z)^4 (xy + yz + xz) (x+y)(y+z) (z+x)}{(x+y+z)^4}
    $$

    With the setting $y=1+z$ this can be simplified to
    $$
    F = frac{(xy + yz + xz) (x+y)(y+z) (z+x)}{(x+y+z)^4}
    $$



    Since by the settings $y=1+z$ and $x = z + (1 +q)/2$ both x and y are linear in z, the numerator of $F$ is of fifth order in $z$, whereas the denominator is of fourth order in $z$. In leading order, the whole term is therefore of first order in $z$ and will therefore rise with $z$ for large enough $z$. This motivates to show that indeed
    $$F(q,z) geq F(q,z=0) = 2 frac{(q+1)^2}{(q+3)^2}$$
    for all $z$ and $q$.
    Showing that directly requires a condition



    $$
    G = (xy + yz + xz) (x+y)(y+z) (z+x) (q+3)^2 - 2 (q+1)^2 (x+y+z)^4 geq 0
    $$



    Inserting $y=1+z$ and $x = z + (1 +q)/2$ into $G$, and expanding the brackets, gives a very lengthy expression which however contains only positive terms, so the condition is proved immediately:



    $$
    G = (z(2q^6z + 2q^6 + 22q^5z^2 + 51q^5z + 22q^5 + 80q^4z^3 + 358q^4z^2 + 357q^4z + 100q^4 + 96q^3z^4 + 960q^3z^3 + 1828q^3z^2 + 1206q^3z + 260q^3 + 864q^2z^4 + 3672q^2z^3 + 4788q^2z^2 + 2484q^2z + 450q^2 + 2592qz^4 + 7344qz^3 + 7398qz^2 + 3159qz + 486q + 2592z^4 + 5832z^3 + 4806z^2 + 1701z + 216))/4 geq 0
    $$



    Hence it suffices, instead of (2), to prove the following:



    $$
    tag{3} frac16 left[ frac{27}{8} right ]^2 2 frac{(q+1)^2}{(q+3)^2} +
    (xy + yz + xz) sum_{cyc} {(x-y)^2}{(x+y)} \ - (x+y)(y+z) (z+x) geqslant 0 quad
    $$



    After insertion of $y=1+z$ and $x = z + (1 +q)/2$, the factors $(x-y)^2$ in the cyclic sum will not be functions of $z$. Hence,
    the LHS is a third order expression in $z$ with leading (in $z^3$ ) term $( 1 + 3 q^2) z^3$, so for large enough $z$ it is rising with $z$. A remarkable feature of this expression is that for the considered range $0leq q leq 1$ it is actually monotonously rising for all $z$. To see this, consider whether there are points with zero slope. The first derivative of the expression with respect to $z$ is



    $$
    q^4/4 + (7q^3z)/2 + (7q^3)/4 + 9q^2z^2 + 9q^2z + (5q^2)/4 - (7qz)/2 - (7q)/4 + 3z^2 + 3z + 1/2
    $$

    Equating this to zero gives
    $$
    z_{1} =
    -(2((13q^6)/4 + (17q^4)/2 + (133q^2)/4 + 3)^{(1/2)} - 7q + 18q^2 + 7q^3 + 6)/(36q^2 + 12)\
    z_{2} =
    -(-2((13q^6)/4 + (17q^4)/2 + (133q^2)/4 + 3)^{(1/2)} - 7q + 18q^2 + 7q^3 + 6)/(36q^2 + 12)
    $$

    One now shows that in the range $0leq q leq 1$, there are only negative solutions $z_{1,2}$. So we will have no zero slopes for $zgeq 0$. Since the polynomials are (roots of) sixth order in q, we investigate the following figures:



    enter image description here



    enter image description here



    enter image description here



    So monotonicity (rising with $z$) is established for all $q$.



    Hence, to show the inequality it suffices to inspect (3) at the smallest $z=0$. This gives



    $$
    ((q + 1)(8q^6 + 88q^5 + 336q^4 + 432q^3 - 216q^2 - 405q + 243))/(64(q + 3)^3) geqslant 0 quad
    $$

    or



    $$
    tag{4} 8q^6 + 88q^5 + 336q^4 + 432q^3 - 216q^2 - 405q + 243 geqslant 0 quad
    $$



    An even stronger requirement is
    $$
    h(q) =
    432q^3 - 216q^2 - 405q + 243 geq 0
    $$

    In the considered range $0leq q leq 1$, $h(q)$ has a minimum which is obtained by taking the first derivative,
    $$
    1296 q^2 - 432 q - 405
    $$

    and equating to zero, which gives $q = 3/4$, and the above $h(q)$ then gives
    $$
    h(q = 3/4) = 0
    $$



    This establishes the inequality. $ qquad Box$






    share|cite|improve this answer











    $endgroup$


















      6












      $begingroup$

      Here's a proof.



      Using Lagrange's identity leads to
      $$sum_{cyc}frac{x}{y+z}=frac32+frac12sum_{cyc}frac{(x-y)^2}{(x+z)(y+z)} qquad (*)$$



      We get this by letting $a = sqrt{x+y}$, $b = sqrt{y+z}$, $c = sqrt{z+x}$,
      $d = frac{1}{sqrt{x+y}}$, $e = frac{1}{sqrt{y+z}}$, $f = frac{1}{sqrt{z+x}}$, and by Lagrange's identity,
      $$(a^2+b^2+c^2)(d^2+e^2+f^2) = (ad+be+cf)^2 + (ae-bd)^2+(af-cd)^2+(bf-ce)^2$$
      which gives
      $$2 (x+y+z)(sum_{cyc}frac{1}{y+z}) = 9 + sum_{cyc} left(frac{sqrt{x+y}}{sqrt{y+z}} - frac{sqrt{y+z}}{sqrt{x+y}}right)^2$$
      or
      $$3 +sum_{cyc}frac{x}{y+z} = frac92 + frac12sum_{cyc} frac{left({x+y} - (y+z)right)^2}{({y+z})(x+y)} $$
      which is the desired equation $(*)$.



      Squaring both sides of the inequality then gives



      $$
      tag{1} frac16 left[ sum_{cyc}frac{(x-y)^2}{(x+z)(y+z)} right ]^2 + sum_{cyc}frac{(x-y)^2}{(x+z)(y+z)} geq frac{(y-z)^2}{xy + yz + xz}
      $$



      We will follow two paths for separate cases.



      Path 1:
      Omitting the square term it suffices to prove
      $$sum_{cyc}frac{(x-y)^2}{(x+z)(y+z)} geqslant frac{(y-z)^2}{xy + yz + xz}$$



      Clearing denominators, we obtain
      $$
      (x-y)^2 (x+y) + (y-z)^2 (y+z) + (z-x)^2 (z+x) geq frac{(y-z)^2}{xy + yz + xz} (x+y) (y+z) (z+x)
      $$



      Using $(x+y)(y+z)(x+z) = (x+y+z)(xy + yz+ xz) - x y z$
      it suffices to show



      $$
      (x-y)^2 (x+y) + (y-z)^2 (y+z) + (z-x)^2 (z+x) geq {(y-z)^2} (x+y+z)
      $$

      or
      $$
      (x-y)^2 (x+y) + (z-x)^2 (z+x) geq (y-z)^2 x
      $$



      Since
      $$
      (y-z)^2 = (y-x + x- z)^2 = (y-x)^2 + (x- z)^2 + 2 (y-x)(x-z)
      $$

      this translates into



      $$
      (y-x)^2 y + (x-z)^2 z geq 2 x(y-x)(x-z)
      $$

      For the two cases $ygeq x ; zgeq x $ and $yleq x ; zleq x $ the RHS $leq 0$ so we are done. For the other two cases, by symmetry, it remains to show the case $y> x ; z < x $.



      Rearranging terms, we can also write
      $$
      (y-x)^3 - (x-z)^3 +x ((y - x) + (z-x))^2 geq 0
      $$



      This holds true at least for $(y-x)^3 geq (x-z)^3$ or $y+zgeq 2 x$.



      So the proof is complete other than for the case $y+z < 2x$ and [ $y> x ; z < x $ or $z> x ; y < x $ ].



      Path 2.



      For the remaining case $y+z < 2 x $ and [$y> x ; z < x$ or $z> x ; y < x $] we will follow a different path. Again, by symmetry, we must inspect only $y+z < 2x$ and $y> x > z$.



      A remark up front: In the following, some high order polynomials of one variable have to be inspected. MATLAB is used, also for plotting behaviours of these polynomials. There is no case in spending effort for further analytical work on polynomials where their behaviour is obvious. Still, what follows contains some "ugly" parts.



      In the squared version (1) of the inequality, we can use a further inequality which has been proved here:
      $$sum_{cyc}frac{(x-y)^2}{(x+z)(y+z)} geqslant frac{27}{8} frac{(y-z)^2}{(x+y+z)^2}$$



      So it suffices to prove



      $$
      frac16 left[ frac{27}{8} frac{(y-z)^2}{(x+y+z)^2} right ]^2 + sum_{cyc}frac{(x-y)^2}{(x+z)(y+z)} geq frac{(y-z)^2}{xy + yz + xz}
      $$



      Some numerical inspection shows immediately that the first term cannot be ommitted.



      Clearing some denominators gives



      $$
      tag{2} frac16 left[ frac{27}{8} right ]^2 frac{(y-z)^4 (xy + yz + xz) (x+y)(y+z) (z+x)}{(x+y+z)^4} + \
      (xy + yz + xz) sum_{cyc} {(x-y)^2}{(x+y)} - (y-z)^2 (x+y)(y+z) (z+x) geqslant 0 quad
      $$



      By homogeneity, we set $y=1+z$.



      The condition $y+z < 2x$ then translates into $1+2z < 2x$, hence we further set $x = z + (1 +q)/2$ where $0leq q leq 1$ since also $x = z + (1 +q)/2 < y = 1 +z$.



      Inserting $y=1+z$ and $x = z + (1 +q)/2$ into (2) is straightforward, the result is lengthy (not displayed here).



      Let us start with focussing on the first term in (2), calling that fraction $F$:
      $$
      F= frac{(y-z)^4 (xy + yz + xz) (x+y)(y+z) (z+x)}{(x+y+z)^4}
      $$

      With the setting $y=1+z$ this can be simplified to
      $$
      F = frac{(xy + yz + xz) (x+y)(y+z) (z+x)}{(x+y+z)^4}
      $$



      Since by the settings $y=1+z$ and $x = z + (1 +q)/2$ both x and y are linear in z, the numerator of $F$ is of fifth order in $z$, whereas the denominator is of fourth order in $z$. In leading order, the whole term is therefore of first order in $z$ and will therefore rise with $z$ for large enough $z$. This motivates to show that indeed
      $$F(q,z) geq F(q,z=0) = 2 frac{(q+1)^2}{(q+3)^2}$$
      for all $z$ and $q$.
      Showing that directly requires a condition



      $$
      G = (xy + yz + xz) (x+y)(y+z) (z+x) (q+3)^2 - 2 (q+1)^2 (x+y+z)^4 geq 0
      $$



      Inserting $y=1+z$ and $x = z + (1 +q)/2$ into $G$, and expanding the brackets, gives a very lengthy expression which however contains only positive terms, so the condition is proved immediately:



      $$
      G = (z(2q^6z + 2q^6 + 22q^5z^2 + 51q^5z + 22q^5 + 80q^4z^3 + 358q^4z^2 + 357q^4z + 100q^4 + 96q^3z^4 + 960q^3z^3 + 1828q^3z^2 + 1206q^3z + 260q^3 + 864q^2z^4 + 3672q^2z^3 + 4788q^2z^2 + 2484q^2z + 450q^2 + 2592qz^4 + 7344qz^3 + 7398qz^2 + 3159qz + 486q + 2592z^4 + 5832z^3 + 4806z^2 + 1701z + 216))/4 geq 0
      $$



      Hence it suffices, instead of (2), to prove the following:



      $$
      tag{3} frac16 left[ frac{27}{8} right ]^2 2 frac{(q+1)^2}{(q+3)^2} +
      (xy + yz + xz) sum_{cyc} {(x-y)^2}{(x+y)} \ - (x+y)(y+z) (z+x) geqslant 0 quad
      $$



      After insertion of $y=1+z$ and $x = z + (1 +q)/2$, the factors $(x-y)^2$ in the cyclic sum will not be functions of $z$. Hence,
      the LHS is a third order expression in $z$ with leading (in $z^3$ ) term $( 1 + 3 q^2) z^3$, so for large enough $z$ it is rising with $z$. A remarkable feature of this expression is that for the considered range $0leq q leq 1$ it is actually monotonously rising for all $z$. To see this, consider whether there are points with zero slope. The first derivative of the expression with respect to $z$ is



      $$
      q^4/4 + (7q^3z)/2 + (7q^3)/4 + 9q^2z^2 + 9q^2z + (5q^2)/4 - (7qz)/2 - (7q)/4 + 3z^2 + 3z + 1/2
      $$

      Equating this to zero gives
      $$
      z_{1} =
      -(2((13q^6)/4 + (17q^4)/2 + (133q^2)/4 + 3)^{(1/2)} - 7q + 18q^2 + 7q^3 + 6)/(36q^2 + 12)\
      z_{2} =
      -(-2((13q^6)/4 + (17q^4)/2 + (133q^2)/4 + 3)^{(1/2)} - 7q + 18q^2 + 7q^3 + 6)/(36q^2 + 12)
      $$

      One now shows that in the range $0leq q leq 1$, there are only negative solutions $z_{1,2}$. So we will have no zero slopes for $zgeq 0$. Since the polynomials are (roots of) sixth order in q, we investigate the following figures:



      enter image description here



      enter image description here



      enter image description here



      So monotonicity (rising with $z$) is established for all $q$.



      Hence, to show the inequality it suffices to inspect (3) at the smallest $z=0$. This gives



      $$
      ((q + 1)(8q^6 + 88q^5 + 336q^4 + 432q^3 - 216q^2 - 405q + 243))/(64(q + 3)^3) geqslant 0 quad
      $$

      or



      $$
      tag{4} 8q^6 + 88q^5 + 336q^4 + 432q^3 - 216q^2 - 405q + 243 geqslant 0 quad
      $$



      An even stronger requirement is
      $$
      h(q) =
      432q^3 - 216q^2 - 405q + 243 geq 0
      $$

      In the considered range $0leq q leq 1$, $h(q)$ has a minimum which is obtained by taking the first derivative,
      $$
      1296 q^2 - 432 q - 405
      $$

      and equating to zero, which gives $q = 3/4$, and the above $h(q)$ then gives
      $$
      h(q = 3/4) = 0
      $$



      This establishes the inequality. $ qquad Box$






      share|cite|improve this answer











      $endgroup$
















        6












        6








        6





        $begingroup$

        Here's a proof.



        Using Lagrange's identity leads to
        $$sum_{cyc}frac{x}{y+z}=frac32+frac12sum_{cyc}frac{(x-y)^2}{(x+z)(y+z)} qquad (*)$$



        We get this by letting $a = sqrt{x+y}$, $b = sqrt{y+z}$, $c = sqrt{z+x}$,
        $d = frac{1}{sqrt{x+y}}$, $e = frac{1}{sqrt{y+z}}$, $f = frac{1}{sqrt{z+x}}$, and by Lagrange's identity,
        $$(a^2+b^2+c^2)(d^2+e^2+f^2) = (ad+be+cf)^2 + (ae-bd)^2+(af-cd)^2+(bf-ce)^2$$
        which gives
        $$2 (x+y+z)(sum_{cyc}frac{1}{y+z}) = 9 + sum_{cyc} left(frac{sqrt{x+y}}{sqrt{y+z}} - frac{sqrt{y+z}}{sqrt{x+y}}right)^2$$
        or
        $$3 +sum_{cyc}frac{x}{y+z} = frac92 + frac12sum_{cyc} frac{left({x+y} - (y+z)right)^2}{({y+z})(x+y)} $$
        which is the desired equation $(*)$.



        Squaring both sides of the inequality then gives



        $$
        tag{1} frac16 left[ sum_{cyc}frac{(x-y)^2}{(x+z)(y+z)} right ]^2 + sum_{cyc}frac{(x-y)^2}{(x+z)(y+z)} geq frac{(y-z)^2}{xy + yz + xz}
        $$



        We will follow two paths for separate cases.



        Path 1:
        Omitting the square term it suffices to prove
        $$sum_{cyc}frac{(x-y)^2}{(x+z)(y+z)} geqslant frac{(y-z)^2}{xy + yz + xz}$$



        Clearing denominators, we obtain
        $$
        (x-y)^2 (x+y) + (y-z)^2 (y+z) + (z-x)^2 (z+x) geq frac{(y-z)^2}{xy + yz + xz} (x+y) (y+z) (z+x)
        $$



        Using $(x+y)(y+z)(x+z) = (x+y+z)(xy + yz+ xz) - x y z$
        it suffices to show



        $$
        (x-y)^2 (x+y) + (y-z)^2 (y+z) + (z-x)^2 (z+x) geq {(y-z)^2} (x+y+z)
        $$

        or
        $$
        (x-y)^2 (x+y) + (z-x)^2 (z+x) geq (y-z)^2 x
        $$



        Since
        $$
        (y-z)^2 = (y-x + x- z)^2 = (y-x)^2 + (x- z)^2 + 2 (y-x)(x-z)
        $$

        this translates into



        $$
        (y-x)^2 y + (x-z)^2 z geq 2 x(y-x)(x-z)
        $$

        For the two cases $ygeq x ; zgeq x $ and $yleq x ; zleq x $ the RHS $leq 0$ so we are done. For the other two cases, by symmetry, it remains to show the case $y> x ; z < x $.



        Rearranging terms, we can also write
        $$
        (y-x)^3 - (x-z)^3 +x ((y - x) + (z-x))^2 geq 0
        $$



        This holds true at least for $(y-x)^3 geq (x-z)^3$ or $y+zgeq 2 x$.



        So the proof is complete other than for the case $y+z < 2x$ and [ $y> x ; z < x $ or $z> x ; y < x $ ].



        Path 2.



        For the remaining case $y+z < 2 x $ and [$y> x ; z < x$ or $z> x ; y < x $] we will follow a different path. Again, by symmetry, we must inspect only $y+z < 2x$ and $y> x > z$.



        A remark up front: In the following, some high order polynomials of one variable have to be inspected. MATLAB is used, also for plotting behaviours of these polynomials. There is no case in spending effort for further analytical work on polynomials where their behaviour is obvious. Still, what follows contains some "ugly" parts.



        In the squared version (1) of the inequality, we can use a further inequality which has been proved here:
        $$sum_{cyc}frac{(x-y)^2}{(x+z)(y+z)} geqslant frac{27}{8} frac{(y-z)^2}{(x+y+z)^2}$$



        So it suffices to prove



        $$
        frac16 left[ frac{27}{8} frac{(y-z)^2}{(x+y+z)^2} right ]^2 + sum_{cyc}frac{(x-y)^2}{(x+z)(y+z)} geq frac{(y-z)^2}{xy + yz + xz}
        $$



        Some numerical inspection shows immediately that the first term cannot be ommitted.



        Clearing some denominators gives



        $$
        tag{2} frac16 left[ frac{27}{8} right ]^2 frac{(y-z)^4 (xy + yz + xz) (x+y)(y+z) (z+x)}{(x+y+z)^4} + \
        (xy + yz + xz) sum_{cyc} {(x-y)^2}{(x+y)} - (y-z)^2 (x+y)(y+z) (z+x) geqslant 0 quad
        $$



        By homogeneity, we set $y=1+z$.



        The condition $y+z < 2x$ then translates into $1+2z < 2x$, hence we further set $x = z + (1 +q)/2$ where $0leq q leq 1$ since also $x = z + (1 +q)/2 < y = 1 +z$.



        Inserting $y=1+z$ and $x = z + (1 +q)/2$ into (2) is straightforward, the result is lengthy (not displayed here).



        Let us start with focussing on the first term in (2), calling that fraction $F$:
        $$
        F= frac{(y-z)^4 (xy + yz + xz) (x+y)(y+z) (z+x)}{(x+y+z)^4}
        $$

        With the setting $y=1+z$ this can be simplified to
        $$
        F = frac{(xy + yz + xz) (x+y)(y+z) (z+x)}{(x+y+z)^4}
        $$



        Since by the settings $y=1+z$ and $x = z + (1 +q)/2$ both x and y are linear in z, the numerator of $F$ is of fifth order in $z$, whereas the denominator is of fourth order in $z$. In leading order, the whole term is therefore of first order in $z$ and will therefore rise with $z$ for large enough $z$. This motivates to show that indeed
        $$F(q,z) geq F(q,z=0) = 2 frac{(q+1)^2}{(q+3)^2}$$
        for all $z$ and $q$.
        Showing that directly requires a condition



        $$
        G = (xy + yz + xz) (x+y)(y+z) (z+x) (q+3)^2 - 2 (q+1)^2 (x+y+z)^4 geq 0
        $$



        Inserting $y=1+z$ and $x = z + (1 +q)/2$ into $G$, and expanding the brackets, gives a very lengthy expression which however contains only positive terms, so the condition is proved immediately:



        $$
        G = (z(2q^6z + 2q^6 + 22q^5z^2 + 51q^5z + 22q^5 + 80q^4z^3 + 358q^4z^2 + 357q^4z + 100q^4 + 96q^3z^4 + 960q^3z^3 + 1828q^3z^2 + 1206q^3z + 260q^3 + 864q^2z^4 + 3672q^2z^3 + 4788q^2z^2 + 2484q^2z + 450q^2 + 2592qz^4 + 7344qz^3 + 7398qz^2 + 3159qz + 486q + 2592z^4 + 5832z^3 + 4806z^2 + 1701z + 216))/4 geq 0
        $$



        Hence it suffices, instead of (2), to prove the following:



        $$
        tag{3} frac16 left[ frac{27}{8} right ]^2 2 frac{(q+1)^2}{(q+3)^2} +
        (xy + yz + xz) sum_{cyc} {(x-y)^2}{(x+y)} \ - (x+y)(y+z) (z+x) geqslant 0 quad
        $$



        After insertion of $y=1+z$ and $x = z + (1 +q)/2$, the factors $(x-y)^2$ in the cyclic sum will not be functions of $z$. Hence,
        the LHS is a third order expression in $z$ with leading (in $z^3$ ) term $( 1 + 3 q^2) z^3$, so for large enough $z$ it is rising with $z$. A remarkable feature of this expression is that for the considered range $0leq q leq 1$ it is actually monotonously rising for all $z$. To see this, consider whether there are points with zero slope. The first derivative of the expression with respect to $z$ is



        $$
        q^4/4 + (7q^3z)/2 + (7q^3)/4 + 9q^2z^2 + 9q^2z + (5q^2)/4 - (7qz)/2 - (7q)/4 + 3z^2 + 3z + 1/2
        $$

        Equating this to zero gives
        $$
        z_{1} =
        -(2((13q^6)/4 + (17q^4)/2 + (133q^2)/4 + 3)^{(1/2)} - 7q + 18q^2 + 7q^3 + 6)/(36q^2 + 12)\
        z_{2} =
        -(-2((13q^6)/4 + (17q^4)/2 + (133q^2)/4 + 3)^{(1/2)} - 7q + 18q^2 + 7q^3 + 6)/(36q^2 + 12)
        $$

        One now shows that in the range $0leq q leq 1$, there are only negative solutions $z_{1,2}$. So we will have no zero slopes for $zgeq 0$. Since the polynomials are (roots of) sixth order in q, we investigate the following figures:



        enter image description here



        enter image description here



        enter image description here



        So monotonicity (rising with $z$) is established for all $q$.



        Hence, to show the inequality it suffices to inspect (3) at the smallest $z=0$. This gives



        $$
        ((q + 1)(8q^6 + 88q^5 + 336q^4 + 432q^3 - 216q^2 - 405q + 243))/(64(q + 3)^3) geqslant 0 quad
        $$

        or



        $$
        tag{4} 8q^6 + 88q^5 + 336q^4 + 432q^3 - 216q^2 - 405q + 243 geqslant 0 quad
        $$



        An even stronger requirement is
        $$
        h(q) =
        432q^3 - 216q^2 - 405q + 243 geq 0
        $$

        In the considered range $0leq q leq 1$, $h(q)$ has a minimum which is obtained by taking the first derivative,
        $$
        1296 q^2 - 432 q - 405
        $$

        and equating to zero, which gives $q = 3/4$, and the above $h(q)$ then gives
        $$
        h(q = 3/4) = 0
        $$



        This establishes the inequality. $ qquad Box$






        share|cite|improve this answer











        $endgroup$



        Here's a proof.



        Using Lagrange's identity leads to
        $$sum_{cyc}frac{x}{y+z}=frac32+frac12sum_{cyc}frac{(x-y)^2}{(x+z)(y+z)} qquad (*)$$



        We get this by letting $a = sqrt{x+y}$, $b = sqrt{y+z}$, $c = sqrt{z+x}$,
        $d = frac{1}{sqrt{x+y}}$, $e = frac{1}{sqrt{y+z}}$, $f = frac{1}{sqrt{z+x}}$, and by Lagrange's identity,
        $$(a^2+b^2+c^2)(d^2+e^2+f^2) = (ad+be+cf)^2 + (ae-bd)^2+(af-cd)^2+(bf-ce)^2$$
        which gives
        $$2 (x+y+z)(sum_{cyc}frac{1}{y+z}) = 9 + sum_{cyc} left(frac{sqrt{x+y}}{sqrt{y+z}} - frac{sqrt{y+z}}{sqrt{x+y}}right)^2$$
        or
        $$3 +sum_{cyc}frac{x}{y+z} = frac92 + frac12sum_{cyc} frac{left({x+y} - (y+z)right)^2}{({y+z})(x+y)} $$
        which is the desired equation $(*)$.



        Squaring both sides of the inequality then gives



        $$
        tag{1} frac16 left[ sum_{cyc}frac{(x-y)^2}{(x+z)(y+z)} right ]^2 + sum_{cyc}frac{(x-y)^2}{(x+z)(y+z)} geq frac{(y-z)^2}{xy + yz + xz}
        $$



        We will follow two paths for separate cases.



        Path 1:
        Omitting the square term it suffices to prove
        $$sum_{cyc}frac{(x-y)^2}{(x+z)(y+z)} geqslant frac{(y-z)^2}{xy + yz + xz}$$



        Clearing denominators, we obtain
        $$
        (x-y)^2 (x+y) + (y-z)^2 (y+z) + (z-x)^2 (z+x) geq frac{(y-z)^2}{xy + yz + xz} (x+y) (y+z) (z+x)
        $$



        Using $(x+y)(y+z)(x+z) = (x+y+z)(xy + yz+ xz) - x y z$
        it suffices to show



        $$
        (x-y)^2 (x+y) + (y-z)^2 (y+z) + (z-x)^2 (z+x) geq {(y-z)^2} (x+y+z)
        $$

        or
        $$
        (x-y)^2 (x+y) + (z-x)^2 (z+x) geq (y-z)^2 x
        $$



        Since
        $$
        (y-z)^2 = (y-x + x- z)^2 = (y-x)^2 + (x- z)^2 + 2 (y-x)(x-z)
        $$

        this translates into



        $$
        (y-x)^2 y + (x-z)^2 z geq 2 x(y-x)(x-z)
        $$

        For the two cases $ygeq x ; zgeq x $ and $yleq x ; zleq x $ the RHS $leq 0$ so we are done. For the other two cases, by symmetry, it remains to show the case $y> x ; z < x $.



        Rearranging terms, we can also write
        $$
        (y-x)^3 - (x-z)^3 +x ((y - x) + (z-x))^2 geq 0
        $$



        This holds true at least for $(y-x)^3 geq (x-z)^3$ or $y+zgeq 2 x$.



        So the proof is complete other than for the case $y+z < 2x$ and [ $y> x ; z < x $ or $z> x ; y < x $ ].



        Path 2.



        For the remaining case $y+z < 2 x $ and [$y> x ; z < x$ or $z> x ; y < x $] we will follow a different path. Again, by symmetry, we must inspect only $y+z < 2x$ and $y> x > z$.



        A remark up front: In the following, some high order polynomials of one variable have to be inspected. MATLAB is used, also for plotting behaviours of these polynomials. There is no case in spending effort for further analytical work on polynomials where their behaviour is obvious. Still, what follows contains some "ugly" parts.



        In the squared version (1) of the inequality, we can use a further inequality which has been proved here:
        $$sum_{cyc}frac{(x-y)^2}{(x+z)(y+z)} geqslant frac{27}{8} frac{(y-z)^2}{(x+y+z)^2}$$



        So it suffices to prove



        $$
        frac16 left[ frac{27}{8} frac{(y-z)^2}{(x+y+z)^2} right ]^2 + sum_{cyc}frac{(x-y)^2}{(x+z)(y+z)} geq frac{(y-z)^2}{xy + yz + xz}
        $$



        Some numerical inspection shows immediately that the first term cannot be ommitted.



        Clearing some denominators gives



        $$
        tag{2} frac16 left[ frac{27}{8} right ]^2 frac{(y-z)^4 (xy + yz + xz) (x+y)(y+z) (z+x)}{(x+y+z)^4} + \
        (xy + yz + xz) sum_{cyc} {(x-y)^2}{(x+y)} - (y-z)^2 (x+y)(y+z) (z+x) geqslant 0 quad
        $$



        By homogeneity, we set $y=1+z$.



        The condition $y+z < 2x$ then translates into $1+2z < 2x$, hence we further set $x = z + (1 +q)/2$ where $0leq q leq 1$ since also $x = z + (1 +q)/2 < y = 1 +z$.



        Inserting $y=1+z$ and $x = z + (1 +q)/2$ into (2) is straightforward, the result is lengthy (not displayed here).



        Let us start with focussing on the first term in (2), calling that fraction $F$:
        $$
        F= frac{(y-z)^4 (xy + yz + xz) (x+y)(y+z) (z+x)}{(x+y+z)^4}
        $$

        With the setting $y=1+z$ this can be simplified to
        $$
        F = frac{(xy + yz + xz) (x+y)(y+z) (z+x)}{(x+y+z)^4}
        $$



        Since by the settings $y=1+z$ and $x = z + (1 +q)/2$ both x and y are linear in z, the numerator of $F$ is of fifth order in $z$, whereas the denominator is of fourth order in $z$. In leading order, the whole term is therefore of first order in $z$ and will therefore rise with $z$ for large enough $z$. This motivates to show that indeed
        $$F(q,z) geq F(q,z=0) = 2 frac{(q+1)^2}{(q+3)^2}$$
        for all $z$ and $q$.
        Showing that directly requires a condition



        $$
        G = (xy + yz + xz) (x+y)(y+z) (z+x) (q+3)^2 - 2 (q+1)^2 (x+y+z)^4 geq 0
        $$



        Inserting $y=1+z$ and $x = z + (1 +q)/2$ into $G$, and expanding the brackets, gives a very lengthy expression which however contains only positive terms, so the condition is proved immediately:



        $$
        G = (z(2q^6z + 2q^6 + 22q^5z^2 + 51q^5z + 22q^5 + 80q^4z^3 + 358q^4z^2 + 357q^4z + 100q^4 + 96q^3z^4 + 960q^3z^3 + 1828q^3z^2 + 1206q^3z + 260q^3 + 864q^2z^4 + 3672q^2z^3 + 4788q^2z^2 + 2484q^2z + 450q^2 + 2592qz^4 + 7344qz^3 + 7398qz^2 + 3159qz + 486q + 2592z^4 + 5832z^3 + 4806z^2 + 1701z + 216))/4 geq 0
        $$



        Hence it suffices, instead of (2), to prove the following:



        $$
        tag{3} frac16 left[ frac{27}{8} right ]^2 2 frac{(q+1)^2}{(q+3)^2} +
        (xy + yz + xz) sum_{cyc} {(x-y)^2}{(x+y)} \ - (x+y)(y+z) (z+x) geqslant 0 quad
        $$



        After insertion of $y=1+z$ and $x = z + (1 +q)/2$, the factors $(x-y)^2$ in the cyclic sum will not be functions of $z$. Hence,
        the LHS is a third order expression in $z$ with leading (in $z^3$ ) term $( 1 + 3 q^2) z^3$, so for large enough $z$ it is rising with $z$. A remarkable feature of this expression is that for the considered range $0leq q leq 1$ it is actually monotonously rising for all $z$. To see this, consider whether there are points with zero slope. The first derivative of the expression with respect to $z$ is



        $$
        q^4/4 + (7q^3z)/2 + (7q^3)/4 + 9q^2z^2 + 9q^2z + (5q^2)/4 - (7qz)/2 - (7q)/4 + 3z^2 + 3z + 1/2
        $$

        Equating this to zero gives
        $$
        z_{1} =
        -(2((13q^6)/4 + (17q^4)/2 + (133q^2)/4 + 3)^{(1/2)} - 7q + 18q^2 + 7q^3 + 6)/(36q^2 + 12)\
        z_{2} =
        -(-2((13q^6)/4 + (17q^4)/2 + (133q^2)/4 + 3)^{(1/2)} - 7q + 18q^2 + 7q^3 + 6)/(36q^2 + 12)
        $$

        One now shows that in the range $0leq q leq 1$, there are only negative solutions $z_{1,2}$. So we will have no zero slopes for $zgeq 0$. Since the polynomials are (roots of) sixth order in q, we investigate the following figures:



        enter image description here



        enter image description here



        enter image description here



        So monotonicity (rising with $z$) is established for all $q$.



        Hence, to show the inequality it suffices to inspect (3) at the smallest $z=0$. This gives



        $$
        ((q + 1)(8q^6 + 88q^5 + 336q^4 + 432q^3 - 216q^2 - 405q + 243))/(64(q + 3)^3) geqslant 0 quad
        $$

        or



        $$
        tag{4} 8q^6 + 88q^5 + 336q^4 + 432q^3 - 216q^2 - 405q + 243 geqslant 0 quad
        $$



        An even stronger requirement is
        $$
        h(q) =
        432q^3 - 216q^2 - 405q + 243 geq 0
        $$

        In the considered range $0leq q leq 1$, $h(q)$ has a minimum which is obtained by taking the first derivative,
        $$
        1296 q^2 - 432 q - 405
        $$

        and equating to zero, which gives $q = 3/4$, and the above $h(q)$ then gives
        $$
        h(q = 3/4) = 0
        $$



        This establishes the inequality. $ qquad Box$







        share|cite|improve this answer














        share|cite|improve this answer



        share|cite|improve this answer








        edited Dec 15 '18 at 21:51

























        answered Jun 19 '16 at 21:28









        AndreasAndreas

        8,2531137




        8,2531137























            4












            $begingroup$

            Since our inequality is symmetric on $y$ and $z$ , we need to check to cases:




            1. $x=min{x,y,z}$.


            Let $y=x+u$ and $z=x+v$. Hence,
            $$4prod_{cyc}(x+y)^2(xy+xz+yz)left(left(sumlimits_{cyc}frac{x}{y+z}right)^2-frac{9}{4}-frac{3(y-z)^2}{2(xy+xz+yz)}right)=$$
            $$=192(u^2+uv+v^2)x^6+48(10u^3+13u^2v+13uv^2+10v^3)x^5+$$
            $$+48(10u^4+17u^3v+18u^2v^2+17uv^3+10v^4)x^4+$$
            $$+4(62u^5+133u^4v+158u^3v^2+158u^2v^3+133uv^4+62v^5)x^3+$$
            $$+(68u^6+192u^5v+225u^2v^4+298u^3v^3+225u^2v^4+192uv^5+68v^6)x^2+$$
            $$+(8u^7+40u^6v+34u^5v^2+66u^4v^3+66u^3v^4+34u^2v^5+40uv^6+8u^7)x+$$
            $$+uv(u+v)^2(4u^4-6u^3v+11u^2v^2-6uv^3+4v^4)geq0;$$
            2. $xneqmin{x,y,z}$.



            Let $x=y+u$ and $z=y+v$. Hence,
            $$4prod_{cyc}(x+y)^2(xy+xz+yz)left(left(sumlimits_{cyc}frac{x}{y+z}right)^2-frac{9}{4}-frac{3(y-z)^2}{2(xy+xz+yz)}right)=$$
            $$=192(3u^2-3uv+v^2)y^6+48(26u^3-3u^2v-19uv^2+10v^3)y^5+$$
            $$+48(22u^4+21u^3v-26u^2v^2-7uv^3+10v^4)y^4+$$
            $$+4(110u^5+253u^4v-58u^3v^2-226u^2v^3+37uv^4+62v^5)y^3+$$
            $$+(92u^6+384u^5v+201u^2v^4-374u^3v^3-231u^2v^4+144uv^5+68v^6)y^2+$$
            $$+2(4u^7+32u^6v+41u^5v^2-15u^4v^3-51u^3v^4-7u^2v^5+20uv^6+4u^7)y+$$
            $$+uv(u+v)^2(4u^4-u^2v^2-6uv^3+4v^4)geq0.$$
            Done!






            share|cite|improve this answer











            $endgroup$













            • $begingroup$
              great proof! by the way are you some olympiad coach?
              $endgroup$
              – vidyarthi
              Jan 31 '17 at 13:06










            • $begingroup$
              @vidyarthi Yes. I one of Israeli team coaches.
              $endgroup$
              – Michael Rozenberg
              Jan 31 '17 at 13:16










            • $begingroup$
              @MichaelRozenberg Typo in the last line of case 1 where $−6uv^3$ appears twice.
              $endgroup$
              – Andreas
              Feb 8 '17 at 10:15










            • $begingroup$
              @MichaelRozenberg Leaves open a number of smaller proofs to establish that all 14 coefficients of the powers of x and y are nonnegative. This is obvious for 5 coefficients which have only positive summands but not obvious for the other 9.
              $endgroup$
              – Andreas
              Feb 8 '17 at 10:15










            • $begingroup$
              @MichaelRozenberg Dear Michael, my best congratulations to you on having reached 10k reputation today! You have contributed tremendously and even accelerated over the last months! Good to have you around! Very best regards, Andreas
              $endgroup$
              – Andreas
              Feb 8 '17 at 18:33


















            4












            $begingroup$

            Since our inequality is symmetric on $y$ and $z$ , we need to check to cases:




            1. $x=min{x,y,z}$.


            Let $y=x+u$ and $z=x+v$. Hence,
            $$4prod_{cyc}(x+y)^2(xy+xz+yz)left(left(sumlimits_{cyc}frac{x}{y+z}right)^2-frac{9}{4}-frac{3(y-z)^2}{2(xy+xz+yz)}right)=$$
            $$=192(u^2+uv+v^2)x^6+48(10u^3+13u^2v+13uv^2+10v^3)x^5+$$
            $$+48(10u^4+17u^3v+18u^2v^2+17uv^3+10v^4)x^4+$$
            $$+4(62u^5+133u^4v+158u^3v^2+158u^2v^3+133uv^4+62v^5)x^3+$$
            $$+(68u^6+192u^5v+225u^2v^4+298u^3v^3+225u^2v^4+192uv^5+68v^6)x^2+$$
            $$+(8u^7+40u^6v+34u^5v^2+66u^4v^3+66u^3v^4+34u^2v^5+40uv^6+8u^7)x+$$
            $$+uv(u+v)^2(4u^4-6u^3v+11u^2v^2-6uv^3+4v^4)geq0;$$
            2. $xneqmin{x,y,z}$.



            Let $x=y+u$ and $z=y+v$. Hence,
            $$4prod_{cyc}(x+y)^2(xy+xz+yz)left(left(sumlimits_{cyc}frac{x}{y+z}right)^2-frac{9}{4}-frac{3(y-z)^2}{2(xy+xz+yz)}right)=$$
            $$=192(3u^2-3uv+v^2)y^6+48(26u^3-3u^2v-19uv^2+10v^3)y^5+$$
            $$+48(22u^4+21u^3v-26u^2v^2-7uv^3+10v^4)y^4+$$
            $$+4(110u^5+253u^4v-58u^3v^2-226u^2v^3+37uv^4+62v^5)y^3+$$
            $$+(92u^6+384u^5v+201u^2v^4-374u^3v^3-231u^2v^4+144uv^5+68v^6)y^2+$$
            $$+2(4u^7+32u^6v+41u^5v^2-15u^4v^3-51u^3v^4-7u^2v^5+20uv^6+4u^7)y+$$
            $$+uv(u+v)^2(4u^4-u^2v^2-6uv^3+4v^4)geq0.$$
            Done!






            share|cite|improve this answer











            $endgroup$













            • $begingroup$
              great proof! by the way are you some olympiad coach?
              $endgroup$
              – vidyarthi
              Jan 31 '17 at 13:06










            • $begingroup$
              @vidyarthi Yes. I one of Israeli team coaches.
              $endgroup$
              – Michael Rozenberg
              Jan 31 '17 at 13:16










            • $begingroup$
              @MichaelRozenberg Typo in the last line of case 1 where $−6uv^3$ appears twice.
              $endgroup$
              – Andreas
              Feb 8 '17 at 10:15










            • $begingroup$
              @MichaelRozenberg Leaves open a number of smaller proofs to establish that all 14 coefficients of the powers of x and y are nonnegative. This is obvious for 5 coefficients which have only positive summands but not obvious for the other 9.
              $endgroup$
              – Andreas
              Feb 8 '17 at 10:15










            • $begingroup$
              @MichaelRozenberg Dear Michael, my best congratulations to you on having reached 10k reputation today! You have contributed tremendously and even accelerated over the last months! Good to have you around! Very best regards, Andreas
              $endgroup$
              – Andreas
              Feb 8 '17 at 18:33
















            4












            4








            4





            $begingroup$

            Since our inequality is symmetric on $y$ and $z$ , we need to check to cases:




            1. $x=min{x,y,z}$.


            Let $y=x+u$ and $z=x+v$. Hence,
            $$4prod_{cyc}(x+y)^2(xy+xz+yz)left(left(sumlimits_{cyc}frac{x}{y+z}right)^2-frac{9}{4}-frac{3(y-z)^2}{2(xy+xz+yz)}right)=$$
            $$=192(u^2+uv+v^2)x^6+48(10u^3+13u^2v+13uv^2+10v^3)x^5+$$
            $$+48(10u^4+17u^3v+18u^2v^2+17uv^3+10v^4)x^4+$$
            $$+4(62u^5+133u^4v+158u^3v^2+158u^2v^3+133uv^4+62v^5)x^3+$$
            $$+(68u^6+192u^5v+225u^2v^4+298u^3v^3+225u^2v^4+192uv^5+68v^6)x^2+$$
            $$+(8u^7+40u^6v+34u^5v^2+66u^4v^3+66u^3v^4+34u^2v^5+40uv^6+8u^7)x+$$
            $$+uv(u+v)^2(4u^4-6u^3v+11u^2v^2-6uv^3+4v^4)geq0;$$
            2. $xneqmin{x,y,z}$.



            Let $x=y+u$ and $z=y+v$. Hence,
            $$4prod_{cyc}(x+y)^2(xy+xz+yz)left(left(sumlimits_{cyc}frac{x}{y+z}right)^2-frac{9}{4}-frac{3(y-z)^2}{2(xy+xz+yz)}right)=$$
            $$=192(3u^2-3uv+v^2)y^6+48(26u^3-3u^2v-19uv^2+10v^3)y^5+$$
            $$+48(22u^4+21u^3v-26u^2v^2-7uv^3+10v^4)y^4+$$
            $$+4(110u^5+253u^4v-58u^3v^2-226u^2v^3+37uv^4+62v^5)y^3+$$
            $$+(92u^6+384u^5v+201u^2v^4-374u^3v^3-231u^2v^4+144uv^5+68v^6)y^2+$$
            $$+2(4u^7+32u^6v+41u^5v^2-15u^4v^3-51u^3v^4-7u^2v^5+20uv^6+4u^7)y+$$
            $$+uv(u+v)^2(4u^4-u^2v^2-6uv^3+4v^4)geq0.$$
            Done!






            share|cite|improve this answer











            $endgroup$



            Since our inequality is symmetric on $y$ and $z$ , we need to check to cases:




            1. $x=min{x,y,z}$.


            Let $y=x+u$ and $z=x+v$. Hence,
            $$4prod_{cyc}(x+y)^2(xy+xz+yz)left(left(sumlimits_{cyc}frac{x}{y+z}right)^2-frac{9}{4}-frac{3(y-z)^2}{2(xy+xz+yz)}right)=$$
            $$=192(u^2+uv+v^2)x^6+48(10u^3+13u^2v+13uv^2+10v^3)x^5+$$
            $$+48(10u^4+17u^3v+18u^2v^2+17uv^3+10v^4)x^4+$$
            $$+4(62u^5+133u^4v+158u^3v^2+158u^2v^3+133uv^4+62v^5)x^3+$$
            $$+(68u^6+192u^5v+225u^2v^4+298u^3v^3+225u^2v^4+192uv^5+68v^6)x^2+$$
            $$+(8u^7+40u^6v+34u^5v^2+66u^4v^3+66u^3v^4+34u^2v^5+40uv^6+8u^7)x+$$
            $$+uv(u+v)^2(4u^4-6u^3v+11u^2v^2-6uv^3+4v^4)geq0;$$
            2. $xneqmin{x,y,z}$.



            Let $x=y+u$ and $z=y+v$. Hence,
            $$4prod_{cyc}(x+y)^2(xy+xz+yz)left(left(sumlimits_{cyc}frac{x}{y+z}right)^2-frac{9}{4}-frac{3(y-z)^2}{2(xy+xz+yz)}right)=$$
            $$=192(3u^2-3uv+v^2)y^6+48(26u^3-3u^2v-19uv^2+10v^3)y^5+$$
            $$+48(22u^4+21u^3v-26u^2v^2-7uv^3+10v^4)y^4+$$
            $$+4(110u^5+253u^4v-58u^3v^2-226u^2v^3+37uv^4+62v^5)y^3+$$
            $$+(92u^6+384u^5v+201u^2v^4-374u^3v^3-231u^2v^4+144uv^5+68v^6)y^2+$$
            $$+2(4u^7+32u^6v+41u^5v^2-15u^4v^3-51u^3v^4-7u^2v^5+20uv^6+4u^7)y+$$
            $$+uv(u+v)^2(4u^4-u^2v^2-6uv^3+4v^4)geq0.$$
            Done!







            share|cite|improve this answer














            share|cite|improve this answer



            share|cite|improve this answer








            edited Feb 8 '17 at 18:37

























            answered Jan 31 '17 at 12:58









            Michael RozenbergMichael Rozenberg

            107k1895199




            107k1895199












            • $begingroup$
              great proof! by the way are you some olympiad coach?
              $endgroup$
              – vidyarthi
              Jan 31 '17 at 13:06










            • $begingroup$
              @vidyarthi Yes. I one of Israeli team coaches.
              $endgroup$
              – Michael Rozenberg
              Jan 31 '17 at 13:16










            • $begingroup$
              @MichaelRozenberg Typo in the last line of case 1 where $−6uv^3$ appears twice.
              $endgroup$
              – Andreas
              Feb 8 '17 at 10:15










            • $begingroup$
              @MichaelRozenberg Leaves open a number of smaller proofs to establish that all 14 coefficients of the powers of x and y are nonnegative. This is obvious for 5 coefficients which have only positive summands but not obvious for the other 9.
              $endgroup$
              – Andreas
              Feb 8 '17 at 10:15










            • $begingroup$
              @MichaelRozenberg Dear Michael, my best congratulations to you on having reached 10k reputation today! You have contributed tremendously and even accelerated over the last months! Good to have you around! Very best regards, Andreas
              $endgroup$
              – Andreas
              Feb 8 '17 at 18:33




















            • $begingroup$
              great proof! by the way are you some olympiad coach?
              $endgroup$
              – vidyarthi
              Jan 31 '17 at 13:06










            • $begingroup$
              @vidyarthi Yes. I one of Israeli team coaches.
              $endgroup$
              – Michael Rozenberg
              Jan 31 '17 at 13:16










            • $begingroup$
              @MichaelRozenberg Typo in the last line of case 1 where $−6uv^3$ appears twice.
              $endgroup$
              – Andreas
              Feb 8 '17 at 10:15










            • $begingroup$
              @MichaelRozenberg Leaves open a number of smaller proofs to establish that all 14 coefficients of the powers of x and y are nonnegative. This is obvious for 5 coefficients which have only positive summands but not obvious for the other 9.
              $endgroup$
              – Andreas
              Feb 8 '17 at 10:15










            • $begingroup$
              @MichaelRozenberg Dear Michael, my best congratulations to you on having reached 10k reputation today! You have contributed tremendously and even accelerated over the last months! Good to have you around! Very best regards, Andreas
              $endgroup$
              – Andreas
              Feb 8 '17 at 18:33


















            $begingroup$
            great proof! by the way are you some olympiad coach?
            $endgroup$
            – vidyarthi
            Jan 31 '17 at 13:06




            $begingroup$
            great proof! by the way are you some olympiad coach?
            $endgroup$
            – vidyarthi
            Jan 31 '17 at 13:06












            $begingroup$
            @vidyarthi Yes. I one of Israeli team coaches.
            $endgroup$
            – Michael Rozenberg
            Jan 31 '17 at 13:16




            $begingroup$
            @vidyarthi Yes. I one of Israeli team coaches.
            $endgroup$
            – Michael Rozenberg
            Jan 31 '17 at 13:16












            $begingroup$
            @MichaelRozenberg Typo in the last line of case 1 where $−6uv^3$ appears twice.
            $endgroup$
            – Andreas
            Feb 8 '17 at 10:15




            $begingroup$
            @MichaelRozenberg Typo in the last line of case 1 where $−6uv^3$ appears twice.
            $endgroup$
            – Andreas
            Feb 8 '17 at 10:15












            $begingroup$
            @MichaelRozenberg Leaves open a number of smaller proofs to establish that all 14 coefficients of the powers of x and y are nonnegative. This is obvious for 5 coefficients which have only positive summands but not obvious for the other 9.
            $endgroup$
            – Andreas
            Feb 8 '17 at 10:15




            $begingroup$
            @MichaelRozenberg Leaves open a number of smaller proofs to establish that all 14 coefficients of the powers of x and y are nonnegative. This is obvious for 5 coefficients which have only positive summands but not obvious for the other 9.
            $endgroup$
            – Andreas
            Feb 8 '17 at 10:15












            $begingroup$
            @MichaelRozenberg Dear Michael, my best congratulations to you on having reached 10k reputation today! You have contributed tremendously and even accelerated over the last months! Good to have you around! Very best regards, Andreas
            $endgroup$
            – Andreas
            Feb 8 '17 at 18:33






            $begingroup$
            @MichaelRozenberg Dear Michael, my best congratulations to you on having reached 10k reputation today! You have contributed tremendously and even accelerated over the last months! Good to have you around! Very best regards, Andreas
            $endgroup$
            – Andreas
            Feb 8 '17 at 18:33




















            draft saved

            draft discarded




















































            Thanks for contributing an answer to Mathematics Stack Exchange!


            • Please be sure to answer the question. Provide details and share your research!

            But avoid



            • Asking for help, clarification, or responding to other answers.

            • Making statements based on opinion; back them up with references or personal experience.


            Use MathJax to format equations. MathJax reference.


            To learn more, see our tips on writing great answers.




            draft saved


            draft discarded














            StackExchange.ready(
            function () {
            StackExchange.openid.initPostLogin('.new-post-login', 'https%3a%2f%2fmath.stackexchange.com%2fquestions%2f1444352%2fstronger-than-nesbitt-inequality%23new-answer', 'question_page');
            }
            );

            Post as a guest















            Required, but never shown





















































            Required, but never shown














            Required, but never shown












            Required, but never shown







            Required, but never shown

































            Required, but never shown














            Required, but never shown












            Required, but never shown







            Required, but never shown







            Popular posts from this blog

            Plaza Victoria

            In PowerPoint, is there a keyboard shortcut for bulleted / numbered list?

            How to put 3 figures in Latex with 2 figures side by side and 1 below these side by side images but in...